K
Khách

Hãy nhập câu hỏi của bạn vào đây, nếu là tài khoản VIP, bạn sẽ được ưu tiên trả lời.

8 tháng 7 2020

\(a^2+b^2+c^2\ge\frac{\left(a+b+c\right)^2}{3}\)

\(\Leftrightarrow3\left(a^2+b^2+c^2\right)\ge\left(a+b+c\right)^2\)

\(\Leftrightarrow3a^2+3b^2+3c^2\ge a^2+b^2+c^2+2ab+2ac+2bc\)

\(\Leftrightarrow2a^2+2b^2+2c^2-2ab-2ac-2bc\ge0\)

\(\Leftrightarrow\left(a-b\right)^2+\left(b-c\right)^2+\left(c-a\right)^2\ge0\)(Luôn đúng)

9 tháng 7 2020

cách khác ạ :3

Áp dụng BĐT Cauchy Schwarz dạng engel ta có :

\(a^2+b^2+c^2=\frac{a^2}{1}+\frac{b^2}{1}+\frac{c^2}{1}\ge\frac{\left(a+b+c\right)^2}{3}\)

Vậy ta có điều phải chứng minh

15 tháng 10 2017

bài 2

(bài này là đề thi olympic Toán,Ireland 1997),nhưng cũng dễ thôi

Giả sử ngược lại \(a^2+b^2+c^2< abc\)

khi đó \(abc>a^2+b^2+c^2>a^2\)nên \(a< bc\)

Tương tự \(b< ac,c< ab\)

Từ đó suy ra :\(a+b+c< ab+bc+ac\left(1\right)\)

mặt khác ta lại có:\(a^2+b^2+c^2\ge ab+bc+ac\)nên

\(abc>a^2+b^2+c^2\ge ab+bc+ac\)

\(\Rightarrow abc>ab+ac+bc\left(2\right)\)

Từ (1),(2) ta có\(abc>a+b+c\)(trái với giả thuyết)

Vậy bài toán được chứng minh

15 tháng 10 2017

3)để đơn giản ta đặt \(x=\frac{1}{a},y=\frac{1}{b},z=\frac{1}{c}\).Khi đó \(x,y,z>0\)

và \(xy+yz+xz\ge1\)

ta phải chứng minh  có ít nhất hai trong ba bất đẳng thức sau đúng

\(2x+3y+6z\ge6,2y+3z+6x\ge6,2z+3x+6y\ge6\)

Giả sử khẳng định này sai,tức là có ít nhất hai trong ba bất đẳng thức trên sai.Không mất tính tổng quát,ta giả sử

\(2x+3y+6z< 6\)và \(2y+3z+6x< 6\)

Cộng hai bất đẳng thức này lại,ta được:\(8x+5y+9z< 12\)

Từ giả thiết \(xy+yz+xz\ge1\Rightarrow x\left(y+z\right)\ge1-yz\)

\(\Rightarrow x\ge\frac{1-yz}{y+z}\)Do đó

\(8\frac{1-yz}{y+z}+5y+9z< 12\Leftrightarrow8\left(1-yz\right)+\left(5y+9z\right)\left(y+z\right)< 12\left(y+z\right)\)

\(\Leftrightarrow5y^2+6yz+9z^2-12y-12z+8< 0\)

\(\Leftrightarrow\left(y+3z-2\right)^2+4\left(y-1\right)^2< 0\)(vô lý)

mâu thuẫn này chứng tỏ khẳng định bài toán đúng.Phép chứng minh hoàn tất.

4 tháng 7 2016

Đặt M; N; P như sau:

\(M=\frac{a^2}{a+b}+\frac{b^2}{b+c}+\frac{c^2}{c+a}\ge N=\frac{c^2}{a+b}+\frac{a^2}{b+c}+\frac{b^2}{c+a}\ge P=\frac{b^2}{a+b}+\frac{c^2}{b+c}+\frac{a^2}{c+a}.\)

1./ Xét hiệu: M - P

\(M-P=\frac{a^2-b^2}{a+b}+\frac{b^2-c^2}{b+c}+\frac{c^2-a^2}{c+a}=a-b+b-c+c-a=0\)

=> M = P

2./ Bất đẳng thức \(M\ge N\ge P\)có \(M=P\)=> \(M=N=P\)

3./ Khi M = N, ta có hiệu: M - N = 0 nên:

\(\frac{a^2-c^2}{a+b}+\frac{b^2-a^2}{b+c}+\frac{c^2-b^2}{c+a}=0\)

\(\Leftrightarrow\frac{\left(a^2-c^2\right)\left(b+c\right)\left(c+a\right)+\left(b^2-a^2\right)\left(a+b\right)\left(a+c\right)+\left(c^2-b^2\right)\left(a+b\right)\left(c+b\right)}{\left(a+b\right)\left(b+c\right)\left(c+a\right)}=0\)

\(\Leftrightarrow a^4+b^4+c^4=a^2b^2+b^2c^2+c^2a^2\)(1)

Mặt khác ta luon có bất đẳng thức: \(\Leftrightarrow a^4+b^4+c^4\ge a^2b^2+b^2c^2+c^2a^2\)dấu "=" khi a2 = b2 = c2

Do đó để xảy ra đẳng thức (1) thì a2 = b2 = c2 hay |a| = |b| = |c|. ĐPCM

4 tháng 7 2016

Làm thì mình nghĩ mình làm dc nhưng có cái giờ phải đi học rồi . Nếu tối nay chưa ai trả lời mình sẽ trả lời 

30 tháng 12 2017

Đề phải cho x,y,z ; a,b,c >0 chứ bạn ơi

Xét A = (a^2/x + b^2/y + c^2/z) . (x+y+z) = [(a/\(\sqrt{x}\))^2+(b/\(\sqrt{y}\))^2+(c/\(\sqrt{z}\))^2 . (\(\sqrt{x}\)2 + \(\sqrt{y}\)2 + \(\sqrt{z}\)2)

Áp dụng bđt bunhiacopxki ta có : 

A >= (a/\(\sqrt{x}\).\(\sqrt{x}\)+b/\(\sqrt{y}\).\(\sqrt{y}\)+c/\(\sqrt{z}\).\(\sqrt{z}\))^2 = (a+b+c)^2

=> a^2/x + b^2/y + c^2/z >= (a+b+c)^2/x+y+z

=> ĐPCM

k mk nha

30 tháng 12 2017

Nhầm chỗ \(\sqrt{z}\)2 nha . đó là \(\sqrt{z}\)2

k mk nha

26 tháng 12 2019

Chuẩn hóa \(a+b+c=3\) rồi dùng hệ số bất định nha bạn.Mình nhác quá chỉ gợi ý thôi.Nếu cần thì trưa mai đi học về mình làm cho.

27 tháng 12 2019

Thấy có lời giải này hay hay nên mình copy lại nha (Trong sách Yếu tố ít nhất - Võ Quốc Bá Cẩn)

13 tháng 6 2017

PP: Dùng tương đương thần chưởng !!!
Ý tưởng : Chứng minh 1/\sqrt{1+a^2} + 1/\sqrt{1+b^2} >= 2/\sqrt{1+ab} >= 2/\sqrt{ 1+ (a+b)^2/4 } 
._. Bạn biết đăng hình ảnh lên đây không mình  làm  ra rùi chụp cho (:

13 tháng 6 2017

BĐT trên chỉ đúng với ab=>1 mà lm gì có ở đề 

16 tháng 8 2017

a,b dể tự làm nha

c)ta có:   \(\left(a-b\right)^2\ge0\Leftrightarrow a^2+b^2-2ab\ge0\Leftrightarrow a^2+2ab+b^2-2ab-2ab\ge0\)

\(\Leftrightarrow\left(a+b\right)^2\ge4ab\)       mà a+b=1

\(\Rightarrow1\ge4ab\Leftrightarrow ab\le\frac{1}{4}\)

lại có: \(\left(a-b\right)^2\ge0\Leftrightarrow a^2+b^2\ge2ab\) mà \(ab\le\frac{1}{4}\)

tahy vào có     \(a^2+b^2\ge2\times\frac{1}{4}\Leftrightarrow a^2+b^2\ge\frac{1}{2}\left(dpcm\right)\)

16 tháng 8 2017

b mình tự làm, bạn làm phần a hộ mình với

22 tháng 11 2016

xét hiệu \(\frac{a^3+b^3}{2}-\left(\frac{a+b}{2}\right)^3=\frac{3}{8}\left(a+b\right)\left(a-b\right)^2\ge0\)(luôn đúng với mọi a,b>0)